Howdy, Stranger!

It looks like you're new here. If you want to get involved, click one of these buttons!

PT14.S4. Q9- Anyone who supports the new tax plan

miriaml7miriaml7 Live Member
edited October 2020 in Logical Reasoning 1023 karma

I have been reading through some of the previous posts regarding this question, and they have really helped me understand the stimulus and why it is wrong. Basically, the author gave us the following premise:

S= support new tax plan
E= chance of being elected
UE= understand economics

P1: S----> /E

P2: UE----> /S

C: E-----> UE

In order for us to get to the author's conclusion (E----- >UE) we should've had the following as our premise:

E-----> /S -----> UE

Valid Argument #3- A--> B--->C

If we had been given this premise, we would have been able to validly get to the author's conclusion.

The author's error is that he is treating UE -----> /S as if it were the same as /S ------> UE. To put it in more simple terms, the author's flaw was a mistaken reversal error (in premise two).

However, I am really confused when it comes to the answer choices. I don't see how answer choice "D" describes the error that has been identified. I have found that for the harder flaw questions, LSAT writers tend to write the answer choices in very convoluted/ abstract manner. It has really been a struggle for me to get past this. Would greatly appreciate someone's help.

Comments

  • canihazJDcanihazJD Alum Member Sage
    edited October 2020 8460 karma

    Premises:

    S→/C
    U→ /S

    therefore:
    C→ U

    The argument is C→/S and (flaw) /S→U

    The argument ignores the possibility that some people who satisfy the necessary condition of the second premise (/S) do not satisfy the sufficient condition (U) or /S←some→/U.

    In other words, it is possible that someone does not support the tax law and also does not understand economics, as opposed to the flawed reversal of the second premise /S→U.

    In other other words, by saying /S→U, the argument is ignoring the possibility of a negation of /S→U (/S←some→/U)

    and AC D gives us that prephrase: Ignores the possibility that /S←some→/U

    The difficulty of this question is because they don't say "mistakes a necessary condition for a sufficient condition", or something like "Mistakes something that is necessary for a particular outcome for something that is merely sufficient for the outcome". Instead it calls out what the flaw is implicitly saying:

    If I say A→B, I am implicitly saying the negation of this statement (A←some→/B) does not happen.

    By saying /S→U (flaw: mistaken reversal) the argument is also saying "not (/S←some→/U)".

    Hope that helps. Great question... those early PTs have some good ones.

  • miriaml7miriaml7 Live Member
    1023 karma

    I appreciate your help! @canihazJD

Sign In or Register to comment.